1. Solve each equation.
a. 5x – 2=8
b. 4x – 3= 2x + 9
C. 6x + 3 = 2x + 8
And show work

Answers

Answer 1

Answer:

a. 5×=8+2

5×=10

b. 4×-2×=9+3

2×=13

c. 6×-2×=8-3

4×=5


Related Questions

Tim has an after-school delivery service that he

provides for several small retailers in town. He

uses his bicycle and charges $1.25 for a delivery

made within 1 mi, $1.70 for a delivery of at

least 1 mi but less than 1 mi, $2.15 for a

delivery of at least 1. mi but less than 2 miles,

and so on. If Tim raised his rates by 10%, what

would he be paid to deliver a package 35

miles?

Answers

Answer:

Step-by-step explanation:

tim has an after school delivery service that he provides for several small retailers in town. he uses his bicycle and charges $1.25 for a delivery made within 1 1/2 miles, $1.70 for a delivery of at least 1 1/2 miles but less than 1 3/4 miles. $2.15 for a delivery of at least 1 3/4 miles but less than 2 miles, and so on. if tim raised his rates by 10%, what would he be paid to deliver a package 3 1/8 miles.

Answer:

From the question asked the cost of additional 1/4 mile (1 3/4 - 1 1/2) is $0.45 ($1.7 - $1.25). If the rate is increased by 10% (0.1), the new price for an additional 1/4 mile would be 1.1 (1 + 0.1) × 0.45 = $0.495.

Tim new charge rate are as follows:

$1.25 for a delivery made within 1 1/2 miles

$1.745 for a delivery of at least 1 1/2 miles but less than 1 3/4

$2.24 or a delivery of at least 1 3/4 miles but less than 2

$2.735 or a delivery of at least 2 miles but less than 2 1/4

$3.23 or a delivery of at least 2 1/4 miles but less than 2 1/2

$3.725 or a delivery of at least 2 1/2 miles but less than 2 3/4

$4.22 or a delivery of at least 2 3/4 miles but less than 3

$4.715 or a delivery of at least 3 miles but less than 3 1/4

Since 3 1/8 is within 3 miles and 3 1/4 miles, Tim would charge $4.715 to deliver a package 3 1/8 miles.

 

PLEASE HELP, WILL GIVE BRAINLIEST IF CORRECT!!!! (08.06 MC) Mike and his friends bought cheese wafers for $2 per packet and chocolate wafers for $1 per packet at a carnival. They spent a total of $25 to buy a total of 20 packets of wafers of the two varieties. Part A: Write a system of equations that can be solved to find the number of packets of cheese wafers and the number of packets of chocolate wafers that Mike and his friends bought at the carnival. Define the variables used in the equations. (5 points) Part B: How many packets of chocolate wafers and cheese wafers did they buy? Explain how you got the answer and why you selected a particular method to get the answer. (5 points)

Answers

Answer:

x = 5 , y = 15

Step-by-step explanation:

You can solve this using substitution.

Let the quantity of cheese wafers be denoted by x and the quantity of chocolate wafers denoted by y

2x + 1y = 25

x + y = 20

These two equations are the answer to part A, (remember to include the above prompt which says what x and y denote).

For part B I used substitution because it was more applicable to the question then addition or elimination.

ACTUAL WORK

Set 2x + 1y = 25 equal to x

x = 25 - y / 2

Replace x with y in the second equation

(25 - y / 2) + y = 20

And solve for y

y = 15

Since we know what y is we can replace y in the second equation and find what x is

x + 15 = 20

Solve for x

x = 5

Answer:

5 Cheese Wafers and 15 Chocolate Wafers

Step-by-step explanation:

please can someone help me solve this.. please help!!​

Answers

Step-by-step explanation:

Hello,

Firstly just look to triangle BDE,

Here, you will find that,

140° = y+80° {the exterior and opposite interior angle of a triangle is equal}.

or, y= 140°-80° {shifting 80° to another side and subtracting it.}

Therefore, the value of y is 60°.

now, let's simply work with line EB or EG. we get;

angle GEF + y=180° { being a linear pair}.

or, angle GEF + 60°= 180°

or, angle GEF = 180°-60°

Therefore, the value of angle GEF = 120°.

now, looking in triangle EFG, we get;

angle GEF + 35°+x= 180° { the sum of interior angle of a triangle is 180°}.

or, 120°+35°+ x= 180°

or, x= 180°- 155°

Therefore, the value of x is 25°.

now, lastly finding the value of "z"

We find that x= z {being vertical opposite angle}

or, z =25°

Therefore, the value of z is 25°.

So, the values are,

x=25°

y=60°

and z= 25°

Hope it helps...

simpily 2^3×3^2=6^5​

Answers

Answer:

2^3×3^2=6^5​  equation is wrong because

2×2×2×3×3=72

6^5=6×6×6×6×6=36×36×6=7776

the two numbers are not equal

Mate, I think your question is wrong ! ;(

[tex]Corrected \\ Question...\\[/tex] (2^3)^2*(3^2)^3=6^5

Solve. 4x−y−2z=−8 −2x+4z=−4 x+2y=6 Enter your answer, in the form (x,y,z), in the boxes in simplest terms. x= y= z=

Answers

Answer:

(-2, 4, 2)

Where x = -2, y = 4, and z = 2.

Step-by-step explanation:

We are given the system of three equations:

[tex]\displaystyle \left\{ \begin{array}{l} 4x -y -2z = -8 \\ -2x + 4z = -4 \\ x + 2y = 6 \end{array}[/tex]

And we want to find the value of each variable.

Note that both the second and third equations have an x.

Therefore, we can isolate the variables for the second and third equation and then substitute them into the first equation to make the first equation all one variable.

Solve the second equation for z:

[tex]\displaystyle \begin{aligned} -2x+4z&=-4 \\ x - 2 &= 2z \\ z&= \frac{x-2}{2}\end{aligned}[/tex]

Likewise, solve the third equation for y:

[tex]\displaystyle \begin{aligned} x+2y &= 6\\ 2y &= 6-x \\ y &= \frac{6-x}{2} \end{aligned}[/tex]

Substitute the above equations into the first:

[tex]\displaystyle 4x - \left(\frac{6-x}{2}\right) - 2\left(\frac{x-2}{2}\right)=-8[/tex]

And solve for x:

[tex]\displaystyle \begin{aligned} 4x+\left(\frac{x-6}{2}\right)+(2-x) &= -8 \\ \\ 8x +(x-6) +(4-2x) &= -16 \\ \\ 7x-2 &= -16 \\ \\ 7x &= -14 \\ \\ x &= -2\end{aligned}[/tex]

Hence, x = -2.

Find z and y using their respective equations:

Second equation:

[tex]\displaystyle \begin{aligned} z&=\frac{x-2}{2} \\ &= \frac{(-2)-2}{2} \\ &= \frac{-4}{2} \\ &= -2\end{aligned}[/tex]

Third equation:

[tex]\displaystyle \begin{aligned} y &= \frac{6-x}{2}\\ &= \frac{6-(-2)}{2}\\ &= \frac{8}{2}\\ &=4\end{aligned}[/tex]

In conclusion, the solution is (-2, 4, -2)

Answer:

x = -2

y =4

z=-2

Step-by-step explanation:

4x−y−2z=−8

−2x+4z=−4

x+2y=6

Solve the second equation for x

x = 6 -2y

Substitute into the first two equations

4x−y−2z=−8

4(6-2y) -y -2 = 8  

24 -8y-y -2z = 8

-9y -2z = -32

−2(6-2y)+4z=−4

-12 +4y +4z = -4

4y+4z = 8

Divide by 4

y+z = 2

z =2-y

Substitute this into -9y -2z = -32

-9y -2(2-y) = -32

-9y -4 +2y = -32

-7y -4 = -32

-7y =-28

y =4

Now find z

z = 2-y

z = 2-4

z = -2

Now find x

x = 6 -2y

x = 6 -2(4)

x =6-8

x = -2

determine the image of the point p[-3,10) under the translation [5,-7]

Answers

[tex](-3+5,10-7)=(2,3)[/tex]

70000000000x50000000000000

Answers

Answer:

Step-by-step explanation: Multiply

70000000000*50000000000000=3.5e+24

How can you change a rational number to a decimal? Can you give an exsample?

Answers

Answer:

1/2=0.5

Step-by-step explanation:

¼=0.25

¾=0.75

need help will give 5 stars.

Answers

Answer:

t=0.64

Step-by-step explanation:

h = -16t^2 +4t +4

We want h =0 since it is hitting the ground

0 = -16t^2 +4t +4

Using the quadratic formula

a = -16  b = 4  c=4

-b ± sqrt( b^2 -4ac)

----------------------------

         2a

-4 ± sqrt( 4^2 -4(-16)4)

----------------------------

         2(-16)

-4 ± sqrt( 16+ 256)

----------------------------

         -32

-4 ± sqrt( 272)

----------------------------

         -32

-4 ± sqrt( 16*17)

----------------------------

         -32

-4 ± sqrt( 16) sqrt(17)

----------------------------

         -32

-4 ± 4 sqrt(17)

----------------------------

         -32

Divide by -4

1 ±  sqrt(17)

----------------------------

         8

To the nearest hundredth

t=-0.39

t=0.64

Since time cannot be negative

t=0.64

Answer:

0.64  

Step-by-step explanation:

0 = -16t^2 + 4t + 4

-4(4t^2 - t -1) = 0

t = [-(-1) +/- sqrt (1 - 4*4*-1)] / 8)

t = 0.64, -0.39

answer is 0.64

Can someone help me with this please it’s algebra 2

Answers

Answer:

7 8 9

Step-by-step explanation:

Evaluate the function below at x=5. Then, enter your solution. f(x)=3(2)^x

Answers

Answer:

Solution: f(5) = 96

Step-by-step explanation:

f(5) = 3(2)^5

f(5) = 3 (2 × 2 × 2 × 2 × 2)

f(5) = 3 (32)

f(5) = 96

If 4SINB=3SIN(2A+B) :
Prove that:7COT(A+B)=COTA

Answers

Answer:

Step-by-step explanation:

Given the expression 4sinB = 3sin(2A+B), we are to show that the expression 7cot(A+B) = cotA

Starting with the expression

4sinB= 3sin(2A+B)

Let us re write angle B = (A + B) - A

and 2A + B = (A + B) + A

Substituting the derived expression back into the original expression ww will have;

4Sin{(A + B) - A } = 3Sin{(A + B)+ A}

From trigonometry identity;

Sin(D+E) = SinDcosE + CosDSinE

Sin(D-E) = SinDcosE - CosDSinE

Applying this in the expression above;

4{Sin(A+B)CosA - Cos(A+B)SinA} = 3{Sin(A+B)CosA + Cos(A+B)sinA}

Open the bracket

4Sin(A+B)CosA - 4Cos(A+B)SinA = 3Sin(A+B)CosA + 3Cos(A+B)sinA

Collecting like terms

4Sin(A+B)CosA - 3Sin(A+B)cosA = 3Cos(A+B)sinA + 4Cos(A+B)sinA

Sin(A+B)CosA = 7Cos(A+B)sinA

Divide both sides by sinA

Sin(A+B)CosA/sinA= 7Cos(A+B)sinA/sinA

Since cosA/sinA = cotA, the expression becomes;

Sin(A+B)cotA = 7Cos(A+B)

Finally, divide both sides of the resulting equation by sin(A+B)

Sin(A+B)cotA/sin(A+B) = 7Cos(A+B)/sin(A+B)

CotA = 7cot(A+B) Proved!

If sin Θ = 5 over 6, what are the values of cos Θ and tan Θ?

Answers

Answer:

Check explanation

Step-by-step explanation:

Sin∅=5/6

Opp=5. Hyp=6

Adj= (√6²+5²)

= √11

Cos∅=(√11)/6

Tan∅=5/(√11)

Answer: cos(theta) = sqrt(11)/6 ; tan(theta)=5/sqrt(11).

Explanation:
sin inverse is restricted to the first and fourth quadrant, so the other side of the triangle is positive sqrt(11) by the Pythagorean theorem.

A combination lock uses three numbers between 1 and 46 with​ repetition, and they must be selected in the correct sequence. Is the name of​ "combination lock"​ appropriate? Why or why​ not? Choose the correct answer below. A. ​No, because the multiplication counting rule would be used to determine the total number of combinations. B. ​Yes, because the combinations rule would be used to determine the total number of combinations. C. ​No, because factorials would be used to determine the total number of combinations. D. ​No, because the permutations rule would be used to determine the total number of combinations.

Answers

The correct answer is D. ​No because the permutations rule would be used to determine the total number of combinations.

Explanation:

The difference between a combination and a permutation is that in permutations the order is considered. This applies to the numbers in a lock because these need to be in order. Therefore, to analyze the permutations in a lock, the rule for permutations should be used. This includes the general formula P (n,r) =[tex]\frac{n!}{(n-r) !}[/tex]; in this, n is the number of objects and r refers to the objects used in a permutation. Thus, the term "combination" is inappropriate because this is a permutation, and the permutation rule should be used.

Solve for x in the equation x squared + 11 x + StartFraction 121 Over 4 EndFraction = StartFraction 125 Over 4 EndFraction.

Answers

Answer:

Below

Step-by-step explanation:

● x^2 + 11x + 121/4 = 125/4

Substract 125/4 from both sides:

● x^2 + 11x + 121/4-125/4= 125/4 -125/4

● x^2 + 11x - (-4/4) = 0

● x^2 +11x -(-1) = 0

● x^2 + 11 x + 1 = 0

This is a quadratic equation so we will use the determinanant (b^2-4ac)

● a = 1

● b = 11

● c = 1

● b^2-4ac = 11^2-4*1*1 = 117

So this equation has two solutions:

● x = (-b -/+ √(b^2-4ac) ) / 2a

● x = (-11 -/+ √(117) ) / 2

● x = (-11 -/+ 3√(13))/ 2

● x = -0.91 or x = -10.9

Round to the nearest unit

● x = -1 or x = -11

The solutions are { -1,-11}

The solution of the equation x² + 11x + (121/4) = 125/4 will be 0.09 and negative 11.09.

What is the solution to the equation?

The distribution of weights to the variables involved that establishes the equilibrium in the calculation is referred to as a result.

The equation is given below.

x² + 11x + (121/4) = 125/4

Simplify the equation, then the equation will be

4x² + 44x + 121 = 125

4x² + 44x + 121 - 125 = 0

4x² + 44x - 4 = 0

x² + 11x - 1 = 0

We know that the formula, then we have

[tex]\rm x = \dfrac{-b \pm \sqrt {b^2 - 4ac}}{2a}[/tex]

The value of a = 1, b = 11, and c = -1. Then we have

[tex]\rm x = \dfrac{-11 \pm \sqrt {11^2 - 4 \times 1 \times (-1)}}{2 \times 1}\\\rm x = \dfrac{-11 \pm \sqrt {121 +4}}{2 }\\x = \dfrac{-11 \pm \sqrt {125}}{2 }[/tex]

Simplify the equation, then we have

x = (- 11 ± 11.18) / 2

x = (-11 - 11.18) / 2, (-11 + 11.18) / 2

x = -11.09, 0.09

The solution of the equation will be 0.09 and negative 11.09.

More about the solution of the equation link is given below.

https://brainly.com/question/545403

#SPJ6

A holiday company charters an aircraft to fly to Malta at
a cost of $22 000. It then sells 150 seats at $185 each and a
futher 35 seats at a 20% discount. Calculate the profit made
per seat if the plane has 200 seats.

Answers

Answer:

$54.65 profit per seat

Step-by-step explanation:

150(185) + 35(185)(.8) = 27,750 + 5,180 = 32,930 - 22,000 = 10,930

10,930/200 = $54.65 profit per seat

Answer:

$54.65

Step-by-step explanation:

First, we find the total amount made. This is easy:

(150 x 185) + (35 x .8(185)) =

27750 + 5180 =

32930

We then subtract the $22000, so the company makes a profit of 10930. There are 200 seats, so the profit made per seat is $54.65

Evaluate the following expression. −8 × (−10) −7× 1/−1

Answers

Answer:

87

Step-by-step explanation:

[tex]-8\left(-10\right)-7 \times \frac{1}{-1}=87\\\\\mathrm{Apply\:rule}\:-\left(-a\right)=a\\\\=8\times \:10-7\times \frac{1}{-1}\\\\8\times \:10=80\\\\7\times \frac{1}{-1}=-7\\\\=80-\left(-7\right)\\\\\mathrm{Apply\:rule}\:-\left(-a\right)=a\\\\=80+7\\\\=87[/tex]

Need help please asap this is not asap but please still give an answer im stuck

Answers

Answer:

135 cubes

Step-by-step explanation:

First, find the volume of the box with the equation V = Bh, where B is the area of the base and h is the height.

V = (2.25)(0.75)(1.25)

V = 2.109375

Next, find the volume of one cube with the side length 1/4 with V = Bh:

V = (0.25)(0.25)(0.25)

V = 0.015625

Then, divide the volume of the box by the volume of one cube:

2.109375 / 0.015625

= 135

The height of a building model is 2% of its actual height. If the building
model is 3 feet tall, how tall is the actual building?

Answers

Answer:

x = 150 feets

Step-by-step explanation:

Given that,

The height of a building model is 2% of its actual height.

The building model is 3 feet tall, h = 3 feet

We need to find the height of the actual building. Let it is x.

According to question,

h = 2% of x

We have, h = 3 feet

So,

[tex]x=\dfrac{h}{2\%}\\\\x=\dfrac{3}{2/100}\\\\x=150\ \text{feet}[/tex]

So, the actual height of the building is 150 feets.

The graph of f(x) = StartRoot x EndRoot is reflected over the y-axis. Use the graphing calculator to graph this reflection. Which list contains three points that lie on the graph of the reflection? (–81, 9), (–36, 6), (–1, 1) (1, –1), (16, –4), (36, –6) (–49, 7), (–18, 9), (–1, 1) (1, –1), (4, –16), (5, –25)

Answers

Answer:

(–81, 9), (–36, 6), (–1, 1) are the correct three points.

Step-by-step explanation:

Given the function:

[tex]f(x) =\sqrt x[/tex]

Please refer to the attached image.

The green line shows the graph of actual function.

It is reflected over y axis.

The reflected graph is shown in black color in attached image.

When reflected over y axis, the sign of variable [tex]x[/tex] changes from Positive to Negative.

So, the resultant function becomes:

[tex]f(x)=\sqrt{-x}[/tex]

i.e. we will have to give the values of x as negative now.

so, the options in which value of x is negative are the possible answers only.

The possible answers are:

(–81, 9), (–36, 6), (–1, 1) and

(–49, 7), (–18, 9), (–1, 1)

Now, we will check the square root function condition.

In the 2nd option, (–18, 9) does not satisfy the condition.

So, the correct answer is:

(–81, 9), (–36, 6), (–1, 1)

Answer:

A on E2020

Step-by-step explanation:

:)

solve this equation -2x+9=-5x-15

Answers

Answer:

x = -8

I hope this helps!

The answer would be x=-8

A bag contains twelve marbles, which includes seven red marbles and five blue marbles. Roja reaches into the bag and pulls out four marbles. a) How many different sets of four marbles can be pulled from this bag? b) How many of these sets contain two red marbles and two blue marbles? c) How many of these sets contain all red marbles? d) How many of these sets contain all red marbles or all blue marbles?

Answers

Answer:

a) 495

b) 210

c) 35

d) 40

Step-by-step explanation:

Given a total of 12 marbles.

n = 12

Number of red marbles = 7

Number of blue marbles = 5

a) Number of different sets of 4 marbles that can be made from this bag ?

This is a simple combination problem.

where n = 12 and r = 4.

So, answer will be:

[tex]_{12}C_4[/tex]

Formula:

[tex]_{n}C_r = \dfrac{n!}{(n-r)!r!}[/tex]

[tex]_{12}C_4 = \dfrac{12!}{(8)!4!} = \dfrac{12\times 11\times 10\times 9}{4 \times 3\times 2} =\bold{495}[/tex]

b) Two red and two blue marbles:

The answer will be:

[tex]_{7}C_2 \times _{5}C_2 = \dfrac{7\times 6}{2} \times \dfrac{5\times 4}{2} =\bold{210}[/tex]

c) all red marbles.(4 chosen out of 7 red and 0 chosen out of 5 blue marbles)

[tex]_{7}C_4 \times _{5}C_0 = \dfrac{7\times 6\times 5\times 4}{4\times 3\times 2} =\bold{35}[/tex]

d) all red or all blue.(all red marbles plus all blue marbles)

All red marbles:

[tex]_{7}C_4 \times _{5}C_0 = \dfrac{7\times 6\times 5\times 4}{4\times 3\times 2} \times 1=\bold{35}[/tex]

All blue marbles:

[tex]_{7}C_0 \times _{5}C_4 = 1 \times \dfrac{5\times 4\times 3\times 2}{4\times 3\times 2} =\bold{5}[/tex]

So, answer is 40.

A rectangular prism has a volume of 864 cubic units. How many cubic unit will fill the volume of the solid if they were packed without any gaps or overlaps

Answers

Answer: 864.

Step-by-step explanation:

The volume of a rectangular prism has a volume equal to:

V = W*L*H

W = width

L = length

H = height

We know that the volume is equal to 864 cubic units.

This means that if we want to fill the prism such that there is no gap or overlap, we should use exactly 864 unit cubes.

Reduce any fractions to lowest terms. Don't round your answer, and don't use mixed fractions. 4x+4\leq9x+84x+4≤9x+8

Answers

Answer:

  x ≥ -4/5

Step-by-step explanation:

Maybe you want to solve ...

  4x+4 ≤ 9x +8

  0 ≤ 5x +4 . . . . . subtract 4x+4

  0 ≤ x +4/5 . . . . . divide by 5

  -4/5 ≤ x . . . . . . . subtract 4/5

Answer:

x ≥−4/5

Step-by-step explanation:

Please answer this question now in two minutes

Answers

Answer:

m∠C = 102°

Step-by-step explanation:

This diagram is a Quadrilateral inscribed in a circle

The first step is to determine what m∠B

is

The sum of opposite angles in an inscribed quadrilateral is equal to 180°

m∠D + m∠B = 180°

m∠B = 180° - m∠D

m∠B = 180° - 80°

m∠B = 100°

Second step is we proceed to determine the exterior angles of the circle

m∠ADC = 2 × m∠B

m∠ADC = 2 × 100°

m∠ADC = 200°

m∠ADC = m∠CD + m∠AD

m∠AD = m∠ADC - m∠CD

m∠AD = 200° - 116°

m∠AD = 84°

The third step is to determine m∠BAD

m∠BAD = m∠AD + m∠AB

m∠BAD = 84° + 120°

m∠BAD = 204°

The final step Is to determine what m∠C is

It is important to note that:

m∠BAD is Opposite m∠C

Hence

m∠C = 1/2 × m∠BAD

m∠C = 1/2 × 204

m∠C = 102°

Jenny had a wardrobe full of 35 different shirts. In order to make more space in her closet, she got rid of 9 of them. What is a reasonable
estimate for the percentage of shirts Jenny got rid of?

Answers

There is no one set answer because there are many ways to estimate here.

35 rounds to 40

9 rounds to 10

She got rid of 10 shirts out of 40, so 10/40 = 1/4 = 0.25 = 25% is the estimated percentage of shirts she got rid of. This is one possible estimate.

Using a calculator, the actual percentage is 9/35 = 0.2571 = 25.71% approximately. So our estimate isn't too bad. Our estimate is an underestimate.

4x=24 solve equation

Answers

Answer:

x=6

Step-by-step explanation:

Rearrange:

Rearrange the equation by subtracting what is to the right of the equal sign from both sides of the equation :

                    4*x-(24)=0

Step by step solution :

STEP

1

:

Pulling out like terms

1.1     Pull out like factors :

  4x - 24  =   4 • (x - 6)

Equation at the end of step

1

:

STEP

2

:

Equations which are never true

2.1      Solve :    4   =  0

This equation has no solution.

A a non-zero constant never equals zero.

Solving a Single Variable Equation:

2.2      Solve  :    x-6 = 0

Add  6  to both sides of the equation :

                     x = 6

One solution was found :

x = 6

Answer:

x= 24/ 4

Step-by-step explanation:

You can simplify it

x= 6/1 which is x= 6

what is the answer to 1/8=s-3/4

Answers

Answer:

7/8 =s

Step-by-step explanation:

1/8=s-3/4

Add 3/4

1/8 + 3/4 = s -3/4 +3/4

1/8 + 3/4 = s

Get a common denominator

1/8 + 3/4 *2/2 = s

1/8 + 6/8 =s

7/8 =s

1/8 = s - 3/4

1/8 = s -6/8 ( * 2/2)

7/8 = s

s = 7/8

I need help factoring this question, Factor 4(20) + 84.

Answers

Answer:

164

Step-by-step explanation:

B for brackets

O for of

D for division

M for multiplication

A for addition

S for subtraction

You first start with the brackets (20) and multiply with 4 which is equal to 80 and then add it to 84 which makes 164

I hope this helps

True or false? If false give counterexample The product of a rational number and an integer is not an integer ​

Answers

Answer:

False

Step-by-step explanation:

Required

State if the product of rational numbers and integer is an integer

The statement is false and the proof is as follows

Literally, rational numbers are decimal numbers that can be represented as a fraction of two integers;

Take for instance: 0.2, 0.5, 2.25, etc.

When any of these numbers is multiplied by an integer, the resulting number can take any of two forms;

1. It can result to an integer:

For instance;

[tex]0.2 * 5 = 1[/tex]

[tex]0.5 * 4 = 2[/tex]

[tex]2.25 * 8 = 18[/tex]

2. It can result in a decimal number

For instance;

[tex]0.2 * 3 = 0.6[/tex]

[tex]0.5 * 5 = 2.5[/tex]

[tex]2.25 * 7 = 15.75[/tex]

From (1) above, we understand that the product can result in an integer.

Hence, the statement is false

Other Questions
Any help will be nice...Dont answer for points >:( Correct equations for vertical line (7.5pt) Correct equations for horizontal lines (7.5pt) 1. In many cities and towns across the United States, the numbering system of the roads is based on a grid, similar to the latitude and longitude lines on a globe. Suppose the green lines in the following graph represent two east-west and two north-south running roads in a Midwestern town. Determine whether each of the following is a characteristic of DNA, RNA, or both.Is single-stranded:Contains nitrogenous bases:Contains uracil:Contains adenine:Is double-stranded:Is made of nucleotides:Contains deoxyribose sugar:DONE According to information found on the production analysis page of the Inquirer, Chester sold 1127 units of Cute in the current year. Assuming that Cute maintains a constant market share, all the units of Cute are sold in the Nano market segment and the growth rate remains constant, how many years will it be before Cute will not be able to meet future demand unless the company adds production capacity ill mark you brainliest, please show work!! Can someone help!!! And explain please Order matters in a combination. TrueFalse what cultural biases about native americans does cabeza de vaca reveal in ""La Relacin"" ? The following shape is based only on squares, semicircles, and quarter circles. Find the area of the shaded part. Which is a theme in Gentleman of Ro en Medio?A-There are different ways to view money, time, and land ownership.B-Rules and regulations determine what the truth is in any situation.C-Land can only be owned by individuals, not an entire community.D-Money and time are valued over anything else by most people. A balloon has an initial volume of 2.954 L containing 5.50 moles of helium. More helium is added so that the balloon expands to 4.325 L. How much helium (moles) has been added if the temperature and pressure stay constant during this process. 10 points :) Graph this for me :P The Jornada was __________. 1.A big debate is raging in both the religious and political arenas regarding same sex marriages. Do you think that if people choose to be committed together, they are entitled to be recognized as a married couple? Why or why not? Present reasons for your answer. 2. Why do you think preventing spousal and domestic abuse should be a concern of the government? Do you think it should? Back up your answers with evidence from the reading. 3.Do you think couples should be entitled to the same legal benefits as those who have a marriage license? Why or why not? 4.Leona Helmsley left her millions to her nine-year-old dog. She was legally sane at the timeand ran her businesses and corporations until the day she died. Do you think she had the legal right to do what she wanted with her money, or was the judge in the case within his rights to change Leonas will? Why or why not? 5. Explain the importance of a will, a power of attorney document, and a guardianship agreement in the case of a cohabitating coupleparents of two childrenwho are killed in an auto accident. PLEASE HELP ASAP will mark the brainiest paragraph if possible A fish jumps out of the water at a speed of 12 feet per second. The height y (in feet) of the fish above the surface of the water is represented by the equation y=-16x^2+12x, where x is the time (in seconds) since the jump began. The fish reaches its highest point above the surface of the water after 0.375 seconds. How far above the surface is the fish at this time? A cabinet door has a perimeter of 76 inches. Its area is 357 square inches. What are the dimensions of the door? What is the domain of f? What is the equivalent annual annuity of a project that requires an investment of $50,000 today and is expected to generate free cash flows of $15,000 per year for the next five years? The companys weighted average cost of capital is 13.1% per year. During the spin cycle of your clothes washer, the tub rotates at a steady angular velocity of 31.7 rad/s. Find the angular displacement of the tub during a spin of 98.3 s, expressed both in radians and in revolutions. Assume that strike Counter has already been declared to be a "pointer to int". Assume further that strike Counter has been initialized -- its value is the address of some int variable. Write a statement that adds 22 to the value of the variable that strikeCounter is pointing to. HELP ASAP WILL ,ARK BRAINLIEST!!! Does the study of geography only have to do with knowing the physical features of the earth and the atmosphere? Why or why not?